Chapter 8 1 The tighter the probability distribution of its expected future 

subject Type Homework Help
subject Pages 14
subject Words 7583
subject Authors Eugene F. Brigham, Joel F. Houston

Unlock document.

This document is partially blurred.
Unlock all pages and 1 million more documents.
Get Access
page-pf1
Chapter 8: Risk and Return True/False Page 263
(Difficulty Levels: Easy, Easy/Medium, Medium, Medium/Hard, and Hard)
Note that there is some overlap between the T/F and the multiple choice questions, as some T/F
statements are used in the MC questions. See the preface for information on the AACSB letter
indicators (F, M, etc.) on the subject lines.
Multiple Choice: True/False
1. The tighter the probability distribution of its expected future returns,
the greater the risk of a given investment as measured by its standard
deviation.
a. True
b. False
2. The coefficient of variation, calculated as the standard deviation of
expected returns divided by the expected return, is a standardized
measure of the risk per unit of expected return.
a. True
b. False
3. The standard deviation is a better measure of risk than the coefficient
of variation if the expected returns of the securities being compared
differ significantly.
a. True
b. False
4. Risk-averse investors require higher rates of return on investments
whose returns are highly uncertain, and most investors are risk averse.
a. True
b. False
5. When adding a randomly chosen new stock to an existing portfolio, the
higher (or more positive) the degree of correlation between the new
stock and stocks already in the portfolio, the less the additional
stock will reduce the portfolio's risk.
a. True
b. False
CHAPTER 8
RISK AND RATES OF RETURN
page-pf2
Page 264 True/False Chapter 8: Risk and Return
6. Diversification will normally reduce the riskiness of a portfolio of
stocks.
a. True
b. False
7. In portfolio analysis, we often use ex post (historical) returns and
standard deviations, despite the fact that we are really interested in
ex ante (future) data.
a. True
b. False
8. The realized return on a stock portfolio is the weighted average of the
expected returns on the stocks in the portfolio.
a. True
b. False
9. Market risk refers to the tendency of a stock to move with the general
stock market. A stock with above-average market risk will tend to be
more volatile than an average stock, and its beta will be greater than
1.0.
a. True
b. False
10. An individual stock's diversifiable risk, which is measured by its beta,
can be lowered by adding more stocks to the portfolio in which the
stock is held.
a. True
b. False
11. Managers should under no conditions take actions that increase their
firm's risk relative to the market, regardless of how much those
actions would increase the firm's expected rate of return.
a. True
b. False
12. One key conclusion of the Capital Asset Pricing Model is that the value
of an asset should be measured by considering both the risk and the
expected return of the asset, assuming that the asset is held in a
well-diversified portfolio. The risk of the asset held in isolation is
not relevant under the CAPM.
a. True
page-pf3
Chapter 8: Risk and Return True/False Page 265
b. False
13. According to the Capital Asset Pricing Model, investors are primarily
concerned with portfolio risk, not the risks of individual stocks held
in isolation. Thus, the relevant risk of a stock is the stock's
contribution to the riskiness of a well-diversified portfolio.
a. True
b. False
14. If investors become less averse to risk, the slope of the Security
Market Line (SML) will increase.
a. True
b. False
15. Most corporations earn returns for their stockholders by acquiring and
operating tangible and intangible assets. The relevant risk of each
asset should be measured in terms of its effect on the risk of the
firm's stockholders.
a. True
b. False
16. Variance is a measure of the variability of returns, and since it
involves squaring the deviation of each actual return from the expected
return, it is always larger than its square root, the standard
deviation.
a. True
b. False
17. Because of differences in the expected returns on different
investments, the standard deviation is not always an adequate measure
of risk. However, the coefficient of variation adjusts for differences
in expected returns and thus allows investors to make better
comparisons of investments' stand-alone risk.
a. True
b. False
18. "Risk aversion" implies that investors require higher expected returns
on riskier than on less risky securities.
a. True
b. False
page-pf4
Page 266 True/False Chapter 8: Risk and Return
19. If investors are risk averse and hold only one stock, we can conclude
that the required rate of return on a stock whose standard deviation is
0.21 will be greater than the required return on a stock whose standard
deviation is 0.10. However, if stocks are held in portfolios, it is
possible that the required return could be higher on the stock with the
lower standard deviation.
a. True
b. False
20. Someone who is risk averse has a general dislike for risk and a
preference for certainty. If risk aversion exists in the market, then
investors in general are willing to accept somewhat lower returns on
less risky securities. Different investors have different degrees of
risk aversion, and the end result is that investors with greater risk
aversion tend to hold securities with lower risk (and therefore a lower
expected return) than investors who have more tolerance for risk.
a. True
b. False
21. A stock's beta measures its diversifiable risk relative to the
diversifiable risks of other firms.
a. True
b. False
22. A stock's beta is more relevant as a measure of risk to an investor who
holds only one stock than to an investor who holds a well-diversified
portfolio.
a. True
b. False
23. If the returns of two firms are negatively correlated, then one of them
must have a negative beta.
a. True
b. False
24. A stock with a beta equal to -1.0 has zero systematic (or market) risk.
a. True
b. False
page-pf5
Chapter 8: Risk and Return True/False Page 267
25. It is possible for a firm to have a positive beta, even if the
correlation between its returns and those of another firm is negative.
a. True
b. False
26. Portfolio A has but one security, while Portfolio B has 100 securities.
Because of diversification effects, we would expect Portfolio B to have
the lower risk. However, it is possible for Portfolio A to be less
risky.
a. True
b. False
27. Portfolio A has but one stock, while Portfolio B consists of all stocks
that trade in the market, each held in proportion to its market value.
Because of its diversification, Portfolio B will by definition be
riskless.
a. True
b. False
28. A portfolio's risk is measured by the weighted average of the standard
deviations of the securities in the portfolio. It is this aspect of
portfolios that allows investors to combine stocks and thus reduce the
riskiness of their portfolios.
a. True
b. False
29. The distributions of rates of return for Companies AA and BB are given
below:
State of the Probability of
Economy This State Occurring AA BB
Boom 0.2 30% -10%
Normal 0.6 10% 5%
Recession 0.2 -5% 50%
We can conclude from the above information that any rational, risk-
averse investor would be better off adding Security AA to a well-
diversified portfolio over Security BB.
a. True
b. False
30. Even if the correlation between the returns on two securities is +1.0,
if the securities are combined in the correct proportions, the
resulting 2-asset portfolio will have less risk than either security
page-pf6
Page 268 True/False Chapter 8: Risk and Return
held alone.
a. True
b. False
31. Bad managerial judgments or unforeseen negative events that happen to a
firm are defined as "company-specific," or "unsystematic," events, and
their effects on investment risk can in theory be diversified away.
a. True
b. False
32. We would generally find that the beta of a single security is more
stable over time than the beta of a diversified portfolio.
a. True
b. False
33. We would almost always find that the beta of a diversified portfolio is
less stable over time than the beta of a single security.
a. True
b. False
34. If an investor buys enough stocks, he or she can, through
diversification, eliminate all of the market risk inherent in owning
stocks, but as a general rule it will not be possible to eliminate all
diversifiable risk.
a. True
b. False
35. The CAPM is built on historic conditions, although in most cases we use
expected future data in applying it. Because betas used in the CAPM
are calculated using expected future data, they are not subject to
changes in future volatility. This is one of the strengths of the
CAPM.
a. True
b. False
36. Under the CAPM, the required rate of return on a firm's common stock is
determined only by the firm's market risk. If its market risk is
known, and if that risk is expected to remain constant, then analysts
have all the information they need to calculate the firm's required rate
of return.
a. True
b. False
page-pf7
Chapter 8: Risk and Return True/False Page 269
37. A firm can change its beta through managerial decisions, including
capital budgeting and capital structure decisions.
a. True
b. False
38. Any change in its beta is likely to affect the required rate of return
on a stock, which implies that a change in beta will likely have an
impact on the stock's price, other things held constant.
a. True
b. False
39. The slope of the SML is determined by the value of beta.
a. True
b. False
40. The slope of the SML is determined by investors' aversion to risk. The
greater the average investor's risk aversion, the steeper the SML.
a. True
b. False
41. If you plotted the returns of a company against those of the market and
found that the slope of your line was negative, the CAPM would indicate
that the required rate of return on the stock should be less than the
risk-free rate for a well-diversified investor, assuming that the
observed relationship is expected to continue in the future.
a. True
b. False
42. If you plotted the returns on a given stock against those of the
market, and if you found that the slope of the regression line was
negative, the CAPM would indicate that the required rate of return on
the stock should be greater than the risk-free rate for a well-
diversified investor, assuming that the observed relationship is
expected to continue into the future.
a. True
b. False
43. The Y-axis intercept of the SML represents the required return of a
portfolio with a beta of zero, which is the risk-free rate.
a. True
page-pf8
Page 270 True/False Chapter 8: Risk and Return
b. False
44. The SML relates required returns to firms' systematic (or market) risk.
The slope and intercept of this line can be influenced by a manager's
actions.
a. True
b. False
45. The Y-axis intercept of the SML indicates the required return on an
individual asset whenever the realized return on an average (b = 1)
stock is zero.
a. True
b. False
46. If the price of money (e.g., interest rates and equity capital costs)
increases due to an increase in anticipated inflation, the risk-free
rate will also increase. If there is no change in investors' risk
aversion, then the market risk premium (rM − rRF) will remain constant.
Also, if there is no change in stocks' betas, then the required rate of
return on each stock as measured by the CAPM will increase by the same
amount as the increase in expected inflation.
a. True
b. False
47. Since the market return represents the expected return on an average
stock, the market return reflects a certain amount of risk. As a
result, there exists a market risk premium, which is the amount over
and above the risk-free rate, that is required to compensate stock
investors for assuming an average amount of risk.
a. True
b. False
48. Assume that two investors each hold a portfolio, and that portfolio is
their only asset. Investor A's portfolio has a beta of minus 2.0,
while Investor B's portfolio has a beta of plus 2.0. Assuming that the
unsystematic risks of the stocks in the two portfolios are the same,
then the two investors face the same amount of risk. However, the
holders of either portfolio could lower their risks, and by exactly the
same amount, by adding some "normal" stocks with beta = 1.0.
a. True
b. False
49. The CAPM is a multi-period model that takes account of differences in
securities' maturities, and it can be used to determine the required
page-pf9
Chapter 8: Risk and Return True/False Page 271
rate of return for any given level of systematic risk.
a. True
b. False
Multiple Choice: Conceptual
50. You have the following data on three stocks:
Stock Standard Deviation Beta
A 20% 0.59
B 10% 0.61
C 12% 1.29
If you are a strict risk minimizer, you would choose Stock ____ if it
is to be held in isolation and Stock ____ if it is to be held as part
of a well-diversified portfolio.
a. A; A.
b. A; B.
c. B; A.
d. C; A.
e. C; B.
51. Which is the best measure of risk for a single asset held in isolation,
and which is the best measure for an asset held in a diversified
portfolio?
a. Variance; correlation coefficient.
b. Standard deviation; correlation coefficient.
c. Beta; variance.
d. Coefficient of variation; beta.
e. Beta; beta.
52. A highly risk-averse investor is considering adding one additional
stock to a 3-stock portfolio, to form a 4-stock portfolio. The three
stocks currently held all have b = 1.0, and they are perfectly
positively correlated with the market. Potential new Stocks A and B
both have expected returns of 15%, are in equilibrium, and are equally
correlated with the market, with r = 0.75. However, Stock A's standard
deviation of returns is 12% versus 8% for Stock B. Which stock should
this investor add to his or her portfolio, or does the choice not
matter?
a. Either A or B, i.e., the investor should be indifferent between the
two.
b. Stock A.
c. Stock B.
d. Neither A nor B, as neither has a return sufficient to compensate for
risk.
e. Add A, since its beta must be lower.
page-pfa
Page 272 Conceptual M/C Chapter 8: Risk and Return
53. Which of the following is NOT a potential problem when estimating and
using betas, i.e., which statement is FALSE?
a. The fact that a security or project may not have a past history that
can be used as the basis for calculating beta.
b. Sometimes, during a period when the company is undergoing a change
such as toward more leverage or riskier assets, the calculated beta
will be drastically different from the "true" or "expected future"
beta.
c. The beta of an "average stock," or "the market," can change over
time, sometimes drastically.
d. Sometimes the past data used to calculate beta do not reflect the
likely risk of the firm for the future because conditions have
changed.
e. The beta coefficient of a stock is normally found by regressing past
returns on a stock against past market returns. This calculated
historical beta may differ from the beta that exists in the future.
54. Which of the following statements is CORRECT?
a. The beta of a portfolio of stocks is always smaller than the betas
of any of the individual stocks.
b. If you found a stock with a zero historical beta and held it as the
only stock in your portfolio, you would by definition have a
riskless portfolio.
c. The beta coefficient of a stock is normally found by regressing past
returns on a stock against past market returns. One could also
construct a scatter diagram of returns on the stock versus those on
the market, estimate the slope of the line of best fit, and use it
as beta. However, this historical beta may differ from the beta
that exists in the future.
d. The beta of a portfolio of stocks is always larger than the betas of
any of the individual stocks.
e. It is theoretically possible for a stock to have a beta of 1.0. If a
stock did have a beta of 1.0, then, at least in theory, its required
rate of return would be equal to the risk-free (default-free) rate
of return, rRF.
page-pfb
Chapter 8: Risk and Return Conceptual M/C Page 273
55. Which of the following statements is CORRECT?
a. Collections Inc. is in the business of collecting past-due accounts
for other companies, i.e., it is a collection agency. Collections'
revenues, profits, and stock price tend to rise during recessions.
This suggests that Collections Inc.'s beta should be quite high, say
2.0, because it does so much better than most other companies when
the economy is weak.
b. Suppose the returns on two stocks are negatively correlated. One
has a beta of 1.2 as determined in a regression analysis using data
for the last 5 years, while the other has a beta of -0.6. The
returns on the stock with the negative beta must have been
negatively correlated with returns on most other stocks during that
5-year period.
c. Suppose you are managing a stock portfolio, and you have information
that leads you to believe the stock market is likely to be very
strong in the immediate future. That is, you are convinced that the
market is about to rise sharply. You should sell your high-beta
stocks and buy low-beta stocks in order to take advantage of the
expected market move.
d. You think that investor sentiment is about to change, and investors
are about to become more risk averse. This suggests that you should
rebalance your portfolio to include more high-beta stocks.
e. If the market risk premium remains constant, but the risk-free rate
declines, then the required returns on low-beta stocks will rise
while those on high-beta stocks will decline.
56. Which of the following statements is CORRECT?
a. If a company with a high beta merges with a low-beta company, the
best estimate of the new merged company's beta is 1.0.
b. Logically, it is easier to estimate the betas associated with
capital budgeting projects than the betas associated with stocks,
especially if the projects are closely associated with research and
development activities.
c. The beta of an "average stock," which is also "the market beta," can
change over time, sometimes drastically.
d. If a newly issued stock does not have a past history that can be
used for calculating beta, then we should always estimate that its
beta will turn out to be 1.0. This is especially true if the
company finances with more debt than the average firm.
e. During a period when a company is undergoing a change such as
increasing its use of leverage or taking on riskier projects, the
calculated historical beta may be drastically different from the
beta that will exist in the future.
57. Stock A's beta is 1.5 and Stock B's beta is 0.5. Which of the
following statements must be true, assuming the CAPM is correct.
a. Stock A would be a more desirable addition to a portfolio then Stock
B.
page-pfc
Page 274 Conceptual M/C Chapter 8: Risk and Return
b. In equilibrium, the expected return on Stock B will be greater than
that on Stock A.
c. When held in isolation, Stock A has more risk than Stock B.
d. Stock B would be a more desirable addition to a portfolio than A.
e. In equilibrium, the expected return on Stock A will be greater than
that on B.
58. Stock X has a beta of 0.5 and Stock Y has a beta of 1.5. Which of the
following statements must be true, according to the CAPM?
a. If you invest $50,000 in Stock X and $50,000 in Stock Y, your 2-
stock portfolio would have a beta significantly lower than 1.0,
provided the returns on the two stocks are not perfectly correlated.
b. Stock Y's realized return during the coming year will be higher than
Stock X's return.
c. If the expected rate of inflation increases but the market risk
premium is unchanged, the required returns on the two stocks should
increase by the same amount.
d. Stock Y's return has a higher standard deviation than Stock X.
e. If the market risk premium declines, but the risk-free rate is
unchanged, Stock X will have a larger decline in its required return
than will Stock Y.
59. You have the following data on (1) the average annual returns of the
market for the past 5 years and (2) similar information on Stocks A and
B. Which of the possible answers best describes the historical betas
for A and B?
Years Market Stock A Stock B
1 0.03 0.16 0.05
2 -0.05 0.20 0.05
3 0.01 0.18 0.05
4 -0.10 0.25 0.05
5 0.06 0.14 0.05
a. bA > 0; bB = 1.
b. bA > +1; bB = 0.
c. bA = 0; bB = -1.
d. bA < 0; bB = 0.
e. bA < -1; bB = 1.
60. Which of the following statements is CORRECT?
a. An investor can eliminate virtually all market risk if he or she
holds a very large and well diversified portfolio of stocks.
b. The higher the correlation between the stocks in a portfolio, the
lower the risk inherent in the portfolio.
c. It is impossible to have a situation where the market risk of a
single stock is less than that of a portfolio that includes the
stock.
d. Once a portfolio has about 40 stocks, adding additional stocks will
not reduce its risk by even a small amount.
page-pfd
Chapter 8: Risk and Return Conceptual M/C Page 275
e. An investor can eliminate virtually all diversifiable risk if he or
she holds a very large, well-diversified portfolio of stocks.
61. Which of the following statements is CORRECT?
a. If you add enough randomly selected stocks to a portfolio, you can
completely eliminate all of the market risk from the portfolio.
b. If you were restricted to investing in publicly traded common
stocks, yet you wanted to minimize the riskiness of your portfolio
as measured by its beta, then according to the CAPM theory you
should invest an equal amount of money in each stock in the market.
That is, if there were 10,000 traded stocks in the world, the least
risky possible portfolio would include some shares of each one.
c. If you formed a portfolio that consisted of all stocks with betas
less than 1.0, which is about half of all stocks, the portfolio
would itself have a beta coefficient that is equal to the weighted
average beta of the stocks in the portfolio, and that portfolio would
have less risk than a portfolio that consisted of all stocks in the
market.
d. Market risk can be eliminated by forming a large portfolio, and if
some Treasury bonds are held in the portfolio, the portfolio can be
made to be completely riskless.
e. A portfolio that consists of all stocks in the market would have a
required return that is equal to the riskless rate.
62. Inflation, recession, and high interest rates are economic events that
are best characterized as being
a. systematic risk factors that can be diversified away.
b. company-specific risk factors that can be diversified away.
c. among the factors that are responsible for market risk.
d. risks that are beyond the control of investors and thus should not be
considered by security analysts or portfolio managers.
e. irrelevant except to governmental authorities like the Federal
Reserve.
63. Which of the following statements is CORRECT?
a. A stock's beta is less relevant as a measure of risk to an investor
with a well-diversified portfolio than to an investor who holds only
that one stock.
b. If an investor buys enough stocks, he or she can, through
diversification, eliminate all of the diversifiable risk inherent in
owning stocks. Therefore, if a portfolio contained all publicly
traded stocks, it would be essentially riskless.
c. The required return on a firm's common stock is, in theory,
determined solely by its market risk. If the market risk is known,
and if that risk is expected to remain constant, then no other
information is required to specify the firm's required return.
d. Portfolio diversification reduces the variability of returns (as
measured by the standard deviation) of each individual stock held in
a portfolio.
page-pfe
Page 276 Conceptual M/C Chapter 8: Risk and Return
e. A security's beta measures its non-diversifiable, or market, risk
relative to that of an average stock.
64. Which of the following statements is CORRECT?
a. A large portfolio of randomly selected stocks will always have a
standard deviation of returns that is less than the standard
deviation of a portfolio with fewer stocks, regardless of how the
stocks in the smaller portfolio are selected.
b. Diversifiable risk can be reduced by forming a large portfolio, but
normally even highly-diversified portfolios are subject to market
(or systematic) risk.
c. A large portfolio of randomly selected stocks will have a standard
deviation of returns that is greater than the standard deviation of
a 1-stock portfolio if that one stock has a beta less than 1.0.
d. A large portfolio of stocks whose betas are greater than 1.0 will
have less market risk than a single stock with a beta = 0.8.
e. If you add enough randomly selected stocks to a portfolio, you can
completely eliminate all of the market risk from the portfolio.
65. Which of the following statements is CORRECT?
a. A two-stock portfolio will always have a lower standard deviation
than a one-stock portfolio.
b. A portfolio that consists of 40 stocks that are not highly
correlated with "the market" will probably be less risky than a
portfolio of 40 stocks that are highly correlated with the market,
assuming the stocks all have the same standard deviations.
c. A two-stock portfolio will always have a lower beta than a one-stock
portfolio.
d. If portfolios are formed by randomly selecting stocks, a 10-stock
portfolio will always have a lower beta than a one-stock portfolio.
e. A stock with an above-average standard deviation must also have an
above-average beta.
66. Consider the following information for three stocks, A, B, and C. The
stocks' returns are positively but not perfectly positively correlated
with one another, i.e., the correlations are all between 0 and 1.
Expected Standard
Stock Return Deviation Beta
A 10% 20% 1.0
B 10% 10% 1.0
C 12% 12% 1.4
Portfolio AB has half of its funds invested in Stock A and half in
Stock B. Portfolio ABC has one third of its funds invested in each of
the three stocks. The risk-free rate is 5%, and the market is in
equilibrium, so required returns equal expected returns. Which of the
following statements is CORRECT?
page-pff
Chapter 8: Risk and Return Conceptual M/C Page 277
a. Portfolio AB has a standard deviation of 20%.
b. Portfolio AB's coefficient of variation is greater than 2.0.
c. Portfolio AB's required return is greater than the required return on
Stock A.
d. Portfolio ABC's expected return is 10.66667%.
e. Portfolio ABC has a standard deviation of 20%.
67. Which of the following statements is CORRECT?
a. If the returns on two stocks are perfectly positively correlated
(i.e., the correlation coefficient is +1.0) and these stocks have
identical standard deviations, an equally weighted portfolio of the
two stocks will have a standard deviation that is less than that of
the individual stocks.
b. A portfolio with a large number of randomly selected stocks would
have more market risk than a single stock that has a beta of 0.5,
assuming that the stock's beta was correctly calculated and is
stable.
c. If a stock has a negative beta, its expected return must be
negative.
d. A portfolio with a large number of randomly selected stocks would
have less market risk than a single stock that has a beta of 0.5.
e. According to the CAPM, stocks with higher standard deviations of
returns must also have higher expected returns.
68. For a portfolio of 40 randomly selected stocks, which of the following
is most likely to be true?
a. The riskiness of the portfolio is greater than the riskiness of each
of the stocks if each was held in isolation.
b. The riskiness of the portfolio is the same as the riskiness of each
stock if it was held in isolation.
c. The beta of the portfolio is less than the weighted average of the
betas of the individual stocks.
d. The beta of the portfolio is equal to the weighted average of the
betas of the individual stocks.
e. The beta of the portfolio is larger than the weighted average of the
betas of the individual stocks.
69. Which of the following statements best describes what you should expect
if you randomly select stocks and add them to your portfolio?
a. Adding more such stocks will reduce the portfolio's unsystematic, or
diversifiable, risk.
b. Adding more such stocks will increase the portfolio's expected rate
of return.
c. Adding more such stocks will reduce the portfolio's beta coefficient
and thus its systematic risk.
d. Adding more such stocks will have no effect on the portfolio's risk.
e. Adding more such stocks will reduce the portfolio's market risk but
not its unsystematic risk.
page-pf10
Page 278 Conceptual M/C Chapter 8: Risk and Return
70. Bob has a $50,000 stock portfolio with a beta of 1.2, an expected
return of 10.8%, and a standard deviation of 25%. Becky also has a
$50,000 portfolio, but it has a beta of 0.8, an expected return of
9.2%, and a standard deviation that is also 25%. The correlation
coefficient, r, between Bob's and Becky's portfolios is zero. If Bob
and Becky marry and combine their portfolios, which of the following
best describes their combined $100,000 portfolio?
a. The combined portfolio's expected return will be less than the simple
weighted average of the expected returns of the two individual
portfolios, 10.0%.
b. The combined portfolio's beta will be equal to a simple weighted
average of the betas of the two individual portfolios, 1.0; its
expected return will be equal to a simple weighted average of the
expected returns of the two individual portfolios, 10.0%; and its
standard deviation will be less than the simple average of the two
portfolios' standard deviations, 25%.
c. The combined portfolio's expected return will be greater than the
simple weighted average of the expected returns of the two
individual portfolios, 10.0%.
d. The combined portfolio's standard deviation will be greater than the
simple average of the two portfolios' standard deviations, 25%.
e. The combined portfolio's standard deviation will be equal to a
simple average of the two portfolios' standard deviations, 25%.
71. Your portfolio consists of $50,000 invested in Stock X and $50,000
invested in Stock Y. Both stocks have an expected return of 15%, betas
of 1.6, and standard deviations of 30%. The returns of the two stocks
are independent, so the correlation coefficient between them, rXY, is
zero. Which of the following statements best describes the
characteristics of your 2-stock portfolio?
a. Your portfolio has a standard deviation of 30%, and its expected
return is 15%.
b. Your portfolio has a standard deviation less than 30%, and its beta
is greater than 1.6.
c. Your portfolio has a beta equal to 1.6, and its expected return is
15%.
d. Your portfolio has a beta greater than 1.6, and its expected return
is greater than 15%.
e. Your portfolio has a standard deviation greater than 30% and a beta
equal to 1.6.
72. Which of the following is most likely to occur as you add randomly
selected stocks to your portfolio, which currently consists of 3
average stocks?
a. The diversifiable risk of your portfolio will likely decline, but the
expected market risk should not change.
b. The expected return of your portfolio is likely to decline.
c. The diversifiable risk will remain the same, but the market risk
will likely decline.
page-pf11
Chapter 8: Risk and Return Conceptual M/C Page 279
d. Both the diversifiable risk and the market risk of your portfolio are
likely to decline.
e. The total risk of your portfolio should decline, and as a result, the
expected rate of return on the portfolio should also decline.
73. Jane has a portfolio of 20 average stocks, and Dick has a portfolio of 2
average stocks. Assuming the market is in equilibrium, which of the
following statements is CORRECT?
a. Jane's portfolio will have less diversifiable risk and also less
market risk than Dick's portfolio.
b. The required return on Jane's portfolio will be lower than that on
Dick's portfolio because Jane's portfolio will have less total risk.
c. Dick's portfolio will have more diversifiable risk, the same market
risk, and thus more total risk than Jane's portfolio, but the
required (and expected) returns will be the same on both portfolios.
d. If the two portfolios have the same beta, their required returns will
be the same, but Jane's portfolio will have less market risk than
Dick's.
e. The expected return on Jane's portfolio must be lower than the
expected return on Dick's portfolio because Jane is more
diversified.
74. Stocks A and B each have an expected return of 12%, a beta of 1.2, and
a standard deviation of 25%. The returns on the two stocks have a
correlation of +0.6. Portfolio P has 50% in Stock A and 50% in Stock
B. Which of the following statements is CORRECT?
a. Portfolio P has a beta that is greater than 1.2.
b. Portfolio P has a standard deviation that is greater than 25%.
c. Portfolio P has an expected return that is less than 12%.
d. Portfolio P has a standard deviation that is less than 25%.
e. Portfolio P has a beta that is less than 1.2.
75. Stocks A, B, and C all have an expected return of 10% and a standard
deviation of 25%. Stocks A and B have returns that are independent of
one another, i.e., their correlation coefficient, r, equals zero.
Stocks A and C have returns that are negatively correlated with one
another, i.e., r is less than 0. Portfolio AB is a portfolio with half
of its money invested in Stock A and half in Stock B. Portfolio AC is
a portfolio with half of its money invested in Stock A and half
invested in Stock C. Which of the following statements is CORRECT?
a. Portfolio AC has an expected return that is less than 10%.
b. Portfolio AC has an expected return that is greater than 25%.
c. Portfolio AB has a standard deviation that is greater than 25%.
d. Portfolio AB has a standard deviation that is equal to 25%.
e. Portfolio AC has a standard deviation that is less than 25%.
76. Stocks A and B each have an expected return of 15%, a standard
deviation of 20%, and a beta of 1.2. The returns on the two stocks have
page-pf12
Page 280 Conceptual M/C Chapter 8: Risk and Return
a correlation coefficient of +0.6. You have a portfolio that consists
of 50% A and 50% B. Which of the following statements is CORRECT?
a. The portfolio's beta is less than 1.2.
b. The portfolio's expected return is 15%.
c. The portfolio's standard deviation is greater than 20%.
d. The portfolio's beta is greater than 1.2.
e. The portfolio's standard deviation is 20%.
77. Stock A has a beta of 0.8, Stock B has a beta of 1.0, and Stock C has a
beta of 1.2. Portfolio P has 1/3 of its value invested in each stock.
Each stock has a standard deviation of 25%, and their returns are
independent of one another, i.e., the correlation coefficients between
each pair of stocks is zero. Assuming the market is in equilibrium,
which of the following statements is CORRECT?
a. Portfolio P's expected return is greater than the expected return on
Stock B.
b. Portfolio P's expected return is equal to the expected return on
Stock A.
c. Portfolio P's expected return is less than the expected return on
Stock B.
d. Portfolio P's expected return is equal to the expected return on
Stock B.
e. Portfolio P's expected return is greater than the expected return on
Stock C.
78. In a portfolio of three randomly selected stocks, which of the following
could NOT be true, i.e., which statement is false?
a. The riskiness of the portfolio is less than the riskiness of each of
the stocks if they were held in isolation.
b. The riskiness of the portfolio is greater than the riskiness of one
or two of the stocks.
c. The beta of the portfolio is lower than the lowest of the three
betas.
d. The beta of the portfolio is higher than the highest of the three
betas.
e. The beta of the portfolio is calculated as a weighted average of the
individual stocks’ betas.
79. Stock A has a beta = 0.8, while Stock B has a beta = 1.6. Which of the
following statements is CORRECT?
a. Stock B's required return is double that of Stock A's.
b. If the marginal investor becomes more risk averse, the required
return on Stock B will increase by more than the required return on
Stock A.
c. An equally weighted portfolio of Stocks A and B will have a beta
lower than 1.2.
page-pf13
Chapter 8: Risk and Return Conceptual M/C Page 281
d. If the marginal investor becomes more risk averse, the required
return on Stock A will increase by more than the required return on
Stock B.
e. If the risk-free rate increases but the market risk premium remains
constant, the required return on Stock A will increase by more than
that on Stock B.
80. Stock A has an expected return of 12%, a beta of 1.2, and a standard
deviation of 20%. Stock B also has a beta of 1.2, but its expected
return is 10% and its standard deviation is 15%. Portfolio AB has
$900,000 invested in Stock A and $300,000 invested in Stock B. The
correlation between the two stocks' returns is zero (that is, rA,B = 0).
Which of the following statements is CORRECT?
a. Portfolio AB's standard deviation is 17.5%.
b. The stocks are not in equilibrium based on the CAPM; if A is valued
correctly, then B is overvalued.
c. The stocks are not in equilibrium based on the CAPM; if A is valued
correctly, then B is undervalued.
d. Portfolio AB's expected return is 11.0%.
e. Portfolio AB's beta is less than 1.2.
81. Stock X has a beta of 0.7 and Stock Y has a beta of 1.3. The standard
deviation of each stock's returns is 20%. The stocks' returns are
independent of each other, i.e., the correlation coefficient, r,
between them is zero. Portfolio P consists of 50% X and 50% Y. Given
this information, which of the following statements is CORRECT?
a. Portfolio P has a standard deviation of 20%.
b. The required return on Portfolio P is equal to the market risk
premium (rM − rRF).
c. Portfolio P has a beta of 0.7.
d. Portfolio P has a beta of 1.0 and a required return that is equal to
the riskless rate, rRF.
e. Portfolio P has the same required return as the market (rM).
82. Which of the following statements is CORRECT? (Assume that the risk-
free rate is a constant.)
a. If the market risk premium increases by 1%, then the required return
will increase for stocks that have a beta greater than 1.0, but it
will decrease for stocks that have a beta less than 1.0.
b. The effect of a change in the market risk premium depends on the
slope of the yield curve.
c. If the market risk premium increases by 1%, then the required return
on all stocks will rise by 1%.
d. If the market risk premium increases by 1%, then the required return
will increase by 1% for a stock that has a beta of 1.0.
e. The effect of a change in the market risk premium depends on the
level of the risk-free rate.
page-pf14
Page 282 Conceptual M/C Chapter 8: Risk and Return
83. Over the past 84 years, we have observed that investments with the
highest average annual returns also tend to have the highest standard
deviations of annual returns. This observation supports the notion
that there is a positive correlation between risk and return. Which of
the following answers correctly ranks investments from highest to
lowest risk (and return), where the security with the highest risk is
shown first, the one with the lowest risk last?
a. Small-company stocks, long-term corporate bonds, large-company
stocks, long-term government bonds, U.S. Treasury bills.
b. Large-company stocks, small-company stocks, long-term corporate
bonds, U.S. Treasury bills, long-term government bonds.
c. Small-company stocks, large-company stocks, long-term corporate
bonds, long-term government bonds, U.S. Treasury bills.
d. U.S. Treasury bills, long-term government bonds, long-term corporate
bonds, small-company stocks, large-company stocks.
e. Large-company stocks, small-company stocks, long-term corporate
bonds, long-term government bonds, U.S. Treasury bills.
84. During the coming year, the market risk premium (rM − rRF), is expected
to fall, while the risk-free rate, rRF, is expected to remain the same.
Given this forecast, which of the following statements is CORRECT?
a. The required return will increase for stocks with a beta less than
1.0 and will decrease for stocks with a beta greater than 1.0.
b. The required return on all stocks will remain unchanged.
c. The required return will fall for all stocks, but it will fall more
for stocks with higher betas.
d. The required return for all stocks will fall by the same amount.
e. The required return will fall for all stocks, but it will fall less
for stocks with higher betas.
85. The risk-free rate is 6%; Stock A has a beta of 1.0; Stock B has a beta
of 2.0; and the market risk premium, rM − rRF, is positive. Which of the
following statements is CORRECT?
a. If the risk-free rate increases but the market risk premium stays
unchanged, Stock B's required return will increase by more than
Stock A's.
b. Stock B's required rate of return is twice that of Stock A.
c. If Stock A's required return is 11%, then the market risk premium is
5%.
d. If Stock B's required return is 11%, then the market risk premium is
5%.
e. If the risk-free rate remains constant but the market risk premium
increases, Stock A's required return will increase by more than
Stock B's.

Trusted by Thousands of
Students

Here are what students say about us.

Copyright ©2022 All rights reserved. | CoursePaper is not sponsored or endorsed by any college or university.